NCLEX type questions with Rationales

Pataasin ang iyong marka sa homework at exams ngayon gamit ang Quizwiz!

Sue is a 34 year old patient who has just been diagnosed with adrenocortical insufficiency. What discharge instructions should be included to manage her corticosteroid therapy? (Select all that apply) a. Diet high in potassium b. Diet high in carbohydrates c. Limit amount exercise to prevent fractures. d. Monitor glucose levels

A, D A. Hypokalemia may develop when taking corticosteroids. A diet high in potassium, protein, and calcium is important. Diet should also be low in fat, and simple carbohydrates. D. Corticosteroids can cause glucose intolerance thus affecting blood glucose levels. It is important to monitor blood glucose levels and educate patient on signs and symptoms of hyperglycemia.

A nurse is administering IV antibiotics for an adult patient recently diagnosed with PKD (Polycystic kidney disease). The patient states, "I can't wait to finish this round of antibiotics so this condition will go away." The nurse plans to re-educate the patient on their diagnosis and includes which of the following (select all that apply): a. This antibiotic is being used to treat your Urinary Tract Infection, not your PKD. b. Treatment of PKD includes antibiotics to prevent UTI infections that can cause additional harm and pain to your kidneys. c. PKD is a life-long condition that will eventually lead to ESRD. d. Kidney transplants are not recommended for patients with PKD.

A, B, C A. PKD does not have a specific rout of treatment. One goal of treatment is the prevention infection of urinary tract, which would be treated with antibiotics. The remaining treatments for PKD are to treat symptoms, like pain, and then begin specific treatments for end-stage renal disease that include nephrectomy, dialysis, and kidney transplant once the disease has progressed. B. The major goal of treating patients with PKD is to prevent urinary tract infections. UTI's can cause additional pain and inflammation. C. PKD usually progresses to ESRD with loss of kidney function. This occurs by the age of 60 in 50% of the population diagnosed with PKD.

A patient with a new ileostomy asks the nurse if they can return immediately to their old diet of 500 ml/day of fluids and junk food. Which of the following are true regarding dietary and fluid management for a new ileostomy? (Select all that apply) a. You will want to drink at least 3000 ml/day worth of fluids to avoid dehydration. b. You will want to identify foods that may cause diarrhea, gas or obstruction. c. Make sure and increase your fluid intake during hot weather, excessive perspiration, and diarrhea to replace lost fluids. d. Fluid and electrolyte deficits are normal for people with ileostomies and do not need to be reported. e. Once your ileostomy starts producing fluids you no longer need to keep a well balanced diet.

A, B, C A. With a new ileostomy, you want to promote fluid intake of AT LEAST 3000 ml/day to avoid dehydration. B. You want to identify foods to avoid to reduce diarrhea, gas, or obstruction. C. With a new ileostomy, you want to increase fluid intake during hot weather, excessive perspiration, and diarrhea to replace losses and prevent dehydration.

Which of the following are risk factors for developing Chronic Kidney Disease? (Select all that apply) a. Diabetes Mellitus b. Hypertension c. Minority population status d. Burns

A, B, C Chronic kidney disease has a high in minority populations, especially African Americans and Native Americans. Although Chronic kidney disease has many different causes, the leading causes are diabetes (about 50%) and hypertension (about 25%).

What are the key characteristics during the compensatory stage of shock? (Select all that apply.) a. Blood is flowing to most essential organs the brain and heart b. Renin-angiotensin system is stimulated due to decreased blood flow to kidneys c. Patient is at risk for developing paralytic ileus d. Sympathetic Nervous System (SNS) releases potent vasoconstrictors (epinephrine and norepinephrine) e. The coronary arteries start to constrict in attempt to meet oxygen demands of myocardium

A, B, C, D A. During the compensatory stage the body tries to overcome consequences of prolonged anaerobic metabolism and attempts to maintain homeostasis. A low blood pressure is due to a decrease in cardiac output and narrowing of pulse pressure. This low blood pressure activates the baroreceptors and vasoconstriction begins. This causes blood to flow to essential organs the brain and the heart. Meanwhile blood is shunted or diverted from remaining organs. The heart needs the blood to meet oxygen demands of the heart. The brain needs blood flow to prevent ischemia and brain damage. B. When there is a decrease of blood flow to the kidneys due to the shunting of the blood the renin-angiotensin system is activated. Renin stimulates the release of angiotensin I which is then converted to angiotensin II. Angiotensin II is a potent constrictor. By causing constriction this increase venous return to the heart increasing pre load, cardiac output, stroke volume and after load. This means the blood pressure is starting to rise. Angiotensin II also stimulates the adrenal cortex to release hormones. Specifically, aldosterone which resorbs sodium and water and excretes potassium. The increase in reabsorbed sodium stimulates the posterior pituitary gland to release antidiuretic hormone which increases water reabsorption. This in return increases blood pressure. C. Blood is shunted from the gastrointestinal tract. This causes impaired motility and slows down peristalsis and consequently the patient is at risk for developing a paralytic ileus. D. The baroreceptors are stimulated due to lack of decreased cardiac output. The baroreceptors are located in the carotid and aortic bodies and respond immediately. They release potent vasoconstrictors epinephrine and norepinephrine. This causes blood to flow to vital organs the brain and heart. Meanwhile blood flow to lungs, kidneys, gastrointestinal tract, and skin are shunted or diverted.

The nurse is providing care for a patient with acute kidney infection, the nurse knows that this patient has an increased risk for which potential complications? (Select all that apply) a. Hyperkalemia b. Neurologic disorders c. Metabolic acidosis d. Respiratory infection

A, B, C, D A. In acute kidney infection, the serum potassium level increases because the kidney's normal ability to excrete potassium is impaired. Hyperkalemia is the serious complication of acute kidney infection because it can cause life-threatening cardiac dysrhythmias. B. Neurologic changes can occur as the nitrogenous waste products accumulate in the brain and other nervous tissues. The manifestations can be as mild as fatigue and difficulty concentrating, and escalate to seizures, stupor, and coma. C. The impaired kidneys cannot excrete hydrogen ions or the acid products of metabolism. A patient with severe metabolic acidosis may develop Kussmaul respirations in an effort to compensate by increasing the exhalation of CO2. D. Leukocytosis is often present with acute kidney infection. The most common cause of death in acute kidney infection is infection. The most common sites of infection are the urinary and respiratory systems.

A patient who is about to receive Radioactive Iodine Therapy is asking you how It works. SELECT ALL THAT APPLY that you would explain to the patient about this process of Iodine Therapy: a. The Iodine is picked up by the thyroid gland b. It may take up to 3 months for full benefit c. The patient will need to follow radiation precautions for a few weeks after therapy d. The Iodine therapy destroys the overactive thyroid cells over time e. The treatment involves making a small incision on the patient's neck

A, B, C, D A. Radioactive Iodine Therapy (RAI) destroys thyroid tissue and in doing so, will then limit the thyroid hormone secretion. It specifically acts on the T3 and T4 and blocks that release into circulation. So, to do so, the RAI has to be picked up into the source which is the thyroid. B. RAI does have a delayed response meaning that although the patient has gotten the dose, they may not get the maximum effect of that dose for up to 3 months. It is important to let the patient know so they understand that if they do not see or feel changes right away, that does not mean they will not eventually. C. RAI patients need to follow radiation precautions and need to understand those precautions. Most of the RAI patients will be treated outpatient so when they are going home they need to know the precautions, so they are not exposing others to their radiation. They need to know to use a private toilet and flush 2-3 times after use. Separate laundry, do not prepare food for others, avoid bring around pregnancy women and children for a minimum of 7 days after therapy. D. Because it is targeting the T3 and T4 cells and it can take up to 3 months, this is a true statement. That it does destroy the overactive thyroid cells over time and in doing so, will stop the undesired symptoms or destroy thyroid cancer cells.

Nursing interventions for management of ascites include...? (Select all that apply) a. Recording daily weights b. Measuring abdominal girth daily c. Administering diuretics d. Rigorous daily exercise e. Low sodium diet

A, B, C, E A. It would be critical to contact the provider if there was a weight gain of greater than 2 pounds for 3 consecutive days. B. It's important to monitor and have a standard measurement of abdominal girth to record buildup of excess fluid in the abdomen. C. By administering diuretics such as spironolactone or furosemide, this allows for a rapid reduction of ascites. E. Sodium restrictive diet will prevent unnecessary fluid retention.

Patrick, a 65-year-old man, has been recently diagnosed with Hyperosmolar hyperglycemic syndrome (HHS). What are some common causes that could have led to the diagnosis of this syndrome? (Select all that apply) a. Urinary tract infection b. Pneumonia c. Sepsis d. Undiagnosed type 1 diabetes e. Newly diagnosed type 2 diabetes

A, B, C, E Common causes of Hyperosmolar hyperglycemic syndrome include urinary tract infections, pneumonia, sepsis, any acute illness, and newly diagnosed type 2 diabetes.

When caring for a colectomy post-op day 2 patient, the nurse becomes concerned the patient has developed sepsis when he starts experiencing which of the following symptoms? (Select all that apply) a. A temperature of 36 degrees Celsius b. An INR of 1.6 c. A HR of 92 beats per minute d. Leukocyte count of 11,000 e. A blood pressure of 92/54

A, B, C, E Diagnostic criteria for sepsis includes temperature > 38.3 degrees Celsius (fever) or < 36 degrees Celsius (hypothermia), heart rate > than 90 beats/minute, respiratory rate >22/min, SBP< 100, blood glucose >140 in a person without diabetes, significant edema, altered mental status, WBC >12,000 or <4,000, elevated C-reactive protein, urine output < 0.5 mL/Kg/hr for 2 hrs, Serum creatine increase >0.5 mg/dL., INR > 1.5 or PTT > 60, platelet count < 100,000, decreased capillary refill or modeling.

When educating the patient, caregiver, and family about incontinent urinary diversion surgery, the nurse understands it is important to include (SELECT ALL THAT APPLY) a. small bowel obstruction is a potential complication after pelvic surgery. b. an assessment of the patient's ability and readiness to learn before beginning. c. observations of mucus in the urine from an ileal conduit should be reported to the healthcare provider as soon as possible. d. the skin around the stoma must be cared for meticulously to prevent dermatitis and yeast infections. e. in this procedure, a patient does not leak involuntarily; there will be no need for an external collecting device.

A, B, D A. After pelvic surgery, there is an increased incidence of thrombophlebitis, small bowel obstruction, and UTI. B. It is important to assess the patient's ability and readiness to learn before initiating a teaching program. Anxiety and fear may interfere with learning. D. Alkaline encrustations with dermatitis may occur when alkaline urine comes in contact with the skin around the stoma; other common problems include yeast infections, product allergies, and shearing-effect excoriations.

A patient comes into the hospital and based on the complete history and physical examination he/she is diagnosed with Acute Poststreptococcal Glomerulonephritis. What are the clinical manifestations that the health care provider is going to see? (Select all that apply) a. Proteinuria b. Hematuria with smoky appearance c. Hyperlipidemia d. Periorbital edema e. Flu-like symptoms

A, B, D A. Depending on the severity of Acute Poststreptococcal Glomerulonephritis, the degree of proteinuria will differ but there will be some no matter what. The dietary protein restriction also varies but the patient will be on a low protein, low sodium, fluid restricted diet. B. Smoky urine in a patient with Acute Poststreptococcal Glomerulonephritis indicates bleeding in the upper urinary tract. The smoky appearance is what distinctly distinguishes Acute Poststreptococcal Glomerulonephritis from another diagnosis. D. Initially edema appears in low pressure tissues, such as those around the eyes (periorbital edema), but later it progresses to involve the total body as ascites or peripheral edema in the legs. The edema is treated by restricting sodium and fluid intake by administering diuretics.

Which of the following are considered nursing interventions for patients going through radiation or chemotherapy? (Select all that apply) a. Monitor BUN and creatinine levels b. Teach patient to avoid large crowds c. Encourage patient to get at least 60 min of direct sunlight per day d. Monitor platelet counts e. High fiber diet to prevent constipation

A, B, D A. Direct renal damage from exposure to nephrotoxic agents such as cisplatin and high dose-methotrexate can be a complication from treatment. Therefore, labs for BUN and creatinine levels must be continuously monitored. B. Infection is a major cause of death in cancer patients because of low WBC's, especially neutrophils. It is recommended that because of immune suppression while undergoing treatment, it is best to avoid large crowds. D. Because spontaneous bleeding can occur with platelet counts <20,000, it is important that the nurse must monitor platelet counts.

Which of the following interventions would be appropriate for a client diagnosed with acute pancreatitis? (Select all that apply) a. Administer pain medications b. Encourage the client to cough and deep breath c. Encourage 3 big meals per day d. Monitor serum calcium levels closely e. Instruct the client that some alcohol intake is okay

A, B, D A. Pain management is a major focus of care because abdominal pain is a prominent symptom of acute pancreatitis. Increased pain can lead to increased metabolic rate and can contribute to further hemodynamic instability. B. Respiratory tract infections are common in acute pancreatitis. This may cause the patient to take shallow and guarded abdominal breaths so utilizing the cough and deep breath technique helps to prevent respiratory tract infections. D. Hypocalcemia is a common imbalance that can occur. Observe the client for tetany (jerking, irritability and muscular twitching) or numbness around the lips or in the fingertips. The client can also be assessed for Chvostek's sign or Trousseau's sign.

A 43 year old male patient has early alcoholic cirrhosis with mild edema in the lower extremities. The nurse explains how to manage cirrhosis and she determines her teaching has been effective when the teach back given by the patient states: (Select all that apply) a. I should stick with a high-carb moderate protein diet b. I should restrict sodium intake c. I should take a specific drug to treat Cirrhosis d. I should be in a semi-fowlers position while in the hospital

A, B, D A. Patients with Cirrhosis should have a diet consistent of high calorie (3000/ day) with high carb-moderate protein. B. The patient with ascites and edema is put on a low sodium diet although the degree of the sodium restriction in based on the patient situation. D. Dyspnea is a frequent problem for patients with severe ascites and can lead to PE. A semi-fowlers position allows for maximal respiratory efficiency, using an air mattress can prevent skin-breakdown.

You are caring for a patient diagnosed with Acute Kidney injury. In educating the patient about this disease, you describe the different types of AKI as prerenal, intrarenal, and postrenal. Which of the following are potential causes of prerenal AKI? Select all that apply. a. Severe dehydration b. Heart failure c. Damage to kidney tissue d. Decreased cardiac output e. Nephrotoxins

A, B, D A. Severe dehydration decreases systemic circulation. The decreased systemic circulation causes a reduction in renal blood flow. The decrease in blood flow leads to decreased glomerular perfusion and filtration of the kidneys. B. Heart failure decreases systemic circulation because the heart is not able to pump effectively. In turn, this failure of the heart to pump is causing circulation issues. The decrease in blood flow leads to decreased glomerular perfusion and filtration of the kidneys. D. Normal adults have cardiac output of 4.7 liters a minute. When CO falls below this amount, there is decreased circulation. The decreased circulation to the kidneys decreases glomerular perfusion and filtration of the kidney.

The nurse is caring for a patient who is complaining of urinary incontinence since the birth of her child. The nurse is educating the patient about Kegel exercises, and knows that teaching was successful if the patient states which of the following? (Select all that apply). a. "These exercises are meant to strengthen the muscles that support my bladder, rectum, vagina, and uterus." b. "I should do both short and long squeezes 40-50 times each day." c. "I should lay on my stomach and squeeze my buttocks together to perform this exercise." d. "When I do the exercise, I imagine I'm on an elevator full of people and I'm trying to control the passing of gas."

A, B, D A. The pelvic floor muscle provides support for your bladder and rectum and, in women, the vagina and the uterus. If this muscle is weak or damaged, problems with the function of the bladder and rectum can occur. B. Short squeezes should consist of contraction of the pelvic floor muscle for about 2 seconds, while long squeezes should consist of contraction for 5-10 seconds. It is suggested that patients should do both of these exercises 40-50 times each day. D. These exercises should also be done without tensing the muscles of your leg, buttocks, or abdomen, but rather by trying to control the passing of gas or pinching off a stool. They suggest that you can imagine you are in an elevator and you feel the urge to pass gas.

A patient's morning lab has hemoglobin level of 5 g/dL, the integumentary changes. The nurse will notice in this patient include the following? (Select all that apply). a. Pallor b. Jaundice c. Anorexia d. Pruritus e. Increased pulse pressure

A, B, D The clinical manifestations of anemia are caused by the body's response to tissue hypoxia. Specific manifestations vary depending on the rate at which the anemia has evolved, its severity, and any coexisting disease. Hemoglobin levels are often used to determine the severity of anemia. The integumentary changes include pallor, jaundice, and pruritus, and in addition to the skin, the sclera of the eyes and mucous membrane are evaluated for jaundice because they reflect the integumentary changes more accurately

The nurse is caring for a patient who had strep throat 8 days ago and is now complaining of flank pain, periorbital edema and said their urine is a rusty color. What lab findings will the nurse expect to see? Select all that apply. a. Proteinuria b. RBCs c. WBSc d. Casts e. Glucose

A, B, D The nurse will suspect that the patient has acute poststreptococcal glomerulonephritis. For this diagnosis, a dipstick urinalysis and urine sediment microscopy would show a significant amount of RBCs, proteinuria that ranges from mild to severe, and erythrocyte casts which are highly indicative of acute poststreptococcal glomerulonephritis. The nurse would also look at the blood tests to see if the blood urea nitrogen (BUN) and serum creatinine were elevated to assess if any renal impairment has occurred.

A patient with a diagnosis of Diabetes Insipidus would experience which of the following fluid and electrolyte imbalance symptoms? Select all that apply. a. Polydipsia b. Polyuria c. Serum sodium 132 mg/dL d. Specific gravity of 1.001 e. Generalized weakness

A, B, D, E A. Diabetes insipidus is caused by deficiency of antidiuretic hormone or decreased response to antidiuretic hormone which will result in increased thirst due to excessive voiding. B. A decreased response or production of antidiuretic hormone will result in the failure of the renal system to retain water and instead release copious amounts. D. Specific gravity will decrease to less than 1.005. Specific gravity measure kidneys ability to concentrate urine. Urine in diabetes insipidus patients will be dilute due to excessive water intake. E. Patient may be tired from nocturia and experience generalized weakness.

What are some of the clinical manifestations of nephrotic syndrome? SELECT ALL THAT APPLY! a. Peripheral edema b. Massive proteinuria c. Hematuria with smoky/rusty appearance d. Hypoalbuminemia e. Hyperlipidemia

A, B, D, E Nephrotic syndrome's clinical manifestations include peripheral edema, massive proteinuria, hypoalbuminemia, hyperlipidemia, hypertension and foamy urine.

In a patient diagnosed with nephrotic syndrome, the nurse would expect to see which characteristic laboratory findings? (Select all that apply). a. decreased serum albumin b. elevated serum cholesterol c. elevated serum creatinine d. hematuria e. decreased total serum protein

A, B, E Nephrotic syndrome results when the glomerulus is excessively permeable to plasma proteins. Characteristic laboratory findings include decreased serum albumin, decreased total serum protein, and elevated serum cholesterol. The increased glomerular membrane permeability found in nephrotic syndrome is responsible for the massive excretion of protein in the urine. The diminished plasma oncotic pressure from the decreased serum proteins stimulates hepatic lipoprotein synthesis, which results in hyperlipidemia. Initially, cholesterol and low-density lipoproteins are elevated and later the triglyceride level is also increased.

A nursing student is teaching a patient about management of constipation. The nursing instructor would intervene if the student made which statement? (Select all that apply) a. The use of laxatives and enemas will help free up blockage. b. Eat foods with high fiber like raw vegetables and beans c. Drink about 3 quarts of fluid including coffee and energy drinks. d. Establish a regular time to defecate. e. Exercise about 3 times a week.

A, C A. Do not overuse laxatives and enemas because they cause dependence. People who overuse them are unable to have a bowel movement without them. C. Fluid soften hard stools. Drink 2 L per day. Drink water or fruit juices. Avoid caffeinated coffee, tea, and cola. Caffeine stimulate fluid loss through urination.

The nurse is providing discharge teaching to a patient who has a new diagnosis of Type 1 Diabetes Mellitus. The nurse understands teaching has been effective when the patient makes which statements? Select all that apply. a. I have an autoimmune disorder b. I will be able to control my disease with diet and exercise only c. My A1C goal is less than 7% d. I will only require a once daily injection of Lantus e. I will carry cookies with me in case my blood glucose drops below 70

A, C A. Type 1 Diabetes Mellitus is an autoimmune disorder in which the body develops antibodies against insulin or the beta pancreatic cells. This results in an 80-90% reduction in islet cells. The person can no longer produce enough insulin to survive. C. A goal of an A1C <7% is the recommended guideline from the ADA. This level decreases the risk for micro and macrovascular damage.

The nurse takes care of a patient recovering from a subtotal thyroidectomy. After successful surgery which nursing interventions should be included in the plan of care? SELECT ALL THAT APPLY. a. Assess the patient every 2 hours for 24 hours for signs of hemorrhage or tracheal compression. b. Administer iodine and B-adrenergic blockers to achieve euthyroid state. c. Place the patient in semi-Fowler's position and support the head with pillows. d. Supply patient with whiteboard and dry erase marker for communication. e. Nurse implements soft diet immediately

A, C, D A. Assess the patient every 2 hours for 24 hours for signs of hemorrhage or tracheal compression, such as irregular breathing, neck swelling, choking or blood on dressing. C. Place patient in semi-fowler's and support neck to avoid flexion of neck and any tension on suture lines. D. Tell the patient that talking is likely to be difficult for a short time after the surgery, patient needs different mode of communication.

A 59-year-old male is admitted to the ER with a distended abdomen thought to be ascites. Ascites is a common manifestation of cirrhosis. What are three mechanisms that lead to ascites? (Select all that apply) a. Hyperaldosteronism b. Inflammation of the sphincter of Oddi associated with cholelithiasis c. Portal hypertension d. Hypoalbuminemia e. Pancreatic abscess

A, C, D A. Hyperaldosteronism, occurs when the hormone aldosterone is metabolized by damaged hepatocytes. The increased level of aldosterone causes increased sodium reabsorption by the renal tubules. This retention of sodium, combined with an increase in antidiuretic hormone in blood, leads to additional water retention. C. Portal hypertension which causes proteins to shift from the blood vessels into the lymph space. When the lymphatic system is unable to carry off the excess proteins and water, they leak into the peritoneal cavity. D. Hypoalbuminemia resulting from the liver's decreased ability to synthesize albumin. The hypoalbuminemia results in decreased colloidal oncotic pressure.

A critically ill patient is diagnosed with severe hypokalemia. Appropriate treatment modalities for the management of severe hypokalemia include (select all that apply) a. Set up continuous electrocardiogram monitoring b. Administer a bolus of intravenous potassium chloride c. Frequently assess the intravenous site for phlebitis and infiltration d. Monitor urine output

A, C, D A. Patients who are critically ill should have continuous electrocardiogram monitoring to detect cardiac changes. C. Intravenous sites used to administer potassium chloride should be assessed for phlebitis and infiltration at least hourly because it is irritating to the vein. Necrosis and sloughing of the surrounding tissue can result from infiltration. D. Potassium chloride is typically only given if urine output meets the parameter of at least 0.5 mL/kg of body weight per hour. Urine output should be monitored appropriately.

A 54-year-old female patient has a suspected infection. Which of the following findings support a diagnosis of sepsis? Select all that apply a. Temperature of 101.3°F b. RBC count 6.2 x 10^12 /L c. Lactate 3 mmol/L d. Heart Rate 110 beats/min e. Respiratory Rate 10/min

A, C, D A. Sepsis is diagnosed when there is a documented or suspected infection, along with general variables such as a temperature greater than 100.9°F (38.3°C). C. Sepsis is diagnosed when there is a documented or suspected infection, along with tissue perfusion variables such as hyperlactatemia. Hyperlactatemia is defined as a lactate level of >1 mmol/L. D. Sepsis is diagnosed when there is a documented or suspected infection, along with general variables such as tachycardia. Tachycardia is defined as heart rate greater than 90 beats/min.

The nurse is getting report on a patient that is in the hospital awaiting laser lithotripsy for their kidney stone. What assessment data do you expect to see in your patient? (Select all that apply) a. Back pain. b. High urine output. c. Cool moist skin. d. Patient frequently changing positions; sitting, standing, walking, laying down.

A, C, D A. The kidney stone pain normally presents as a sharp pain in the flank/back area. C. The patient may be in mild shock with cool, moist skin. D. Patients with renal colic have a hard time staying still because they cannot get comfortable, a common name for this is the kidney stone dance.

When hemodialysis is performed, the nurse will want to closely observe the patient for a decrease in blood pressure that will manifest itself in which way(s). (Select all that apply.) a. vision changes b. increases cardiovascular disease c. seizure activity d. nausea/vomiting

A, C, D The drop in blood pressure during dialysis may precipitate light-headedness, nausea, vomiting, seizures, vision changes, and chest pain from cardiac ischemia.

A nurse is reviewing lab values for a patient admitted to the hospital for acute pancreatitis, what lab values would the nurse expect to see? (Select all that apply.) a. Elevated Serum Amylase b. Decreased Phosphorus c. Decreased Calcium d. Elevated Serum Lipase

A, C, D The primary diagnostic tests for acute pancreatitis are serum amylase and lipase. The serum amylase level is usually elevated early and remains elevated for 24 to 72 hours.

When providing education about diet for a patient who has had a history of gout and urinary calculi, the nurse should tell the patient to avoid or limit which of the following food selections (SELECT ALL THAT APPLY) a. Liver b. Corn c. Mussels d. Salmon e. Tomatoes

A, C, D The treatment for a patient with a history of uric acid urinary calculi is reduce urinary concentration of uric acid. Alkalinize urine with potassium citrate. Administer allopurinol. Reduce dietary purines. Also, foods high in purine include sardines, herring, mussels, liver, kidney, goose, goose, venison, meat soups, sweet breads. Foods moderate in purine; chicken, salmon, crab, veal, mutton, bacon, pork, beef, ham.

Which statement(s) is/are correct about Acute Kidney Injury (AKI)? Select all that apply. a. The most common intrarenal cause is acute tubular necrosis. b. The onset is gradual and often over many years. c. Prerenal and postrenal AKI usually resolves quickly with treatment. d. The RIFLE classification is used to describe the stages of AKI. e. Diagnostic criteria of AKI are acute reduction in urine output and /or elevation in serum creatinine.

A, C, D, E A. Acute tubular necrosis (ATN) is the most common intrarenal cause of AKI and is primarily the result of ischemia, nephrotoxins, or sepsis. C. Prerenal and postrenal AKI that has not caused intrarenal damage usually resolves quickly with treatment. When parenchymal damage occurs due to either prerenal or postrenal causes, or when parenchymal damage occurs directly as with intrarenal causes, AKI has a prolonged course. D. The RIFLE classification is used to describe the stages of AKI. Risk, Injury, Failure, Loss, and End-stage renal disease. E. Although changes in urine output and serum creatinine occur relatively late in the course of AKI, they are known diagnostic indicators. The rate of increase in serum creatinine is also important as a diagnostic indicator in determining the severity of injury.

The nurse is caring for a patient who develops septic shock. The nurse knows to take which of the following actions? (Select all that apply). a. Administer large amounts of isotonic crystalloid fluid replacement. b. Start antibiotic therapy and then obtain cultures. c. Maintain glucose levels below 180 mg/dL. d. Perform a cardiac catheterization as soon as possible. e. Monitor vital signs, especially temperature.

A, C, E A. Patients in septic shock require large amounts of fluid replacement. Volume resuscitation of 30 to 50 mL/kg is usually done with isotonic crystalloids to achieve adequate fluid resuscitation. C. Glucose levels should be maintained below 180 mg/dL (10.0 mmol/L) for patients in shock... Monitor glucose levels in all patients in septic shock according to agency policy. E. Septic shock: focused physical examination including vital signs, cardiopulmonary assessment, capillary refill, peripheral pulses, and skin...Monitor temperature.

A 24-year-old female is admitted to the hospital for sickle cell crisis. She is ready to discharge and the nurse is preparing her discharge education. What should the nurse include in her teaching? (Select all that apply) a. Avoid areas of high elevation b. Use of home oxygen therapy c. Genetic testing for her and her partner d. Maintain adequate fluid intake e. Treat infections promptly

A, C, D, E A. Areas of high altitude have less oxygen supply in the air. The most common trigger of a sickling episode is low oxygen in the blood. It is important that the patient avoids high altitudes when possible in order to prevent the sickling of red blood cells. C. The patient is at an age where starting a family may be in her near future. It is important that she understands the genetic link with sickle cell disease and the likelihood of it being passed to any future children she may have. The likelihood is increased if her partner has the gene as well. D. Dehydration puts the patient at risk for a sickling episode. Fluids and electrolytes will help reduce blood viscosity and maintain kidney function. E. Viral or bacterial infection can lead to hypoxia and trigger a sickling episode. Infection, such as chronic leg ulcers, is also a frequent complication of sickle cell disease. It is important that the patient receives treatment immediately for any infection so a sickling episode does not occur.

A nurse is coming onto their shift and is receiving report on a 76-year-old male patient who arrived on the floor yesterday with dysuria, general abdominal pain and white blood cells in his urine. What signs, if seen, may indicate that emergent care is needed?Select all that apply. a. Patient, who was alert and oriented yesterday, is now confused and agitated b. Patient has a HR of 65, which has been stable since the beginning of her stay c. The patient's hands are warm and mottled d. The patient has a lactate level of 2.0 which is up from 0.75 yesterday e. Patient has had no urine output in the last 12 hours

A, C, D, E A. The patient was admitted with symptoms linked to a urinary tract infection, the change in level of consciousness and mental status over a day is a sign that the patient is in the compensatory stage of septic shock. This sign is an indicator that emergent care is needed for this patient. C. In septic shock the inflammatory response caused by the exaggerated immune response to a microorganism leads to damage to the endothelium, vasodilation and increased capillary permeability which culminate in warm and mottled hands and feet. This is a sign of the compensatory stage of septic shock and emergent care is indicated. D. Lactic acid is a by-product of anaerobic metabolism which is caused by hypoperfusion and impaired oxygen consumption at the cellular level. This is due to a lack of regular blood flow seen in septic shock. In sepsis the lactate level will be increased and the fact that her level is above the normal range and elevated from her labs yesterday indicate that the patient is in septic shock and emergent care is indicated. E. In the compensatory stage of septic shock, reduced blood flow activates renin which stimulates angiotensin to generate angiotensin I which than converts to angiotensin II. Angiotensin II stimulates the adrenal cortex to emit aldosterone which amplifies sodium and water reabsorption in the kidneys. The rise in sodium stimulates the release of antidiuretic hormone which further promotes the reabsorption of water. All of this causes the patient to have low to no urine output. Emergent care is indicated in this patient.

The nurse understands that the first manifestation(s) of Polycystic Kidney Disease (PKD) is/are: (Select all that apply) a. Hypertension b. Elevated Triglycerides c. Headaches d. Hematuria e. Pain

A, C, D, E Early in the disease of PKD, patients are generally asymptomatic. Diagnosis is based on clinical manifestations, family history, ultrasound, and computed tomography. Hypertension is one of the early manifestations at the time of diagnosis. The patient does not necessarily have to be in end stage renal failure each time they are having clinical symptoms, this presentation is based on the above diagnostic criteria. However, as PKD advances, PKD and End Stage Renal Disease (ESRD) do share some of the same manifestations as well as nursing management measures; they are two different diseases.

Important nursing interventions when caring for a patient with SIADH include (Select all that apply) a. Fluid restriction b. Increase fluids c. Administer a loop diuretic d. Initiate seizure and fall precautions e. Elevate the HOB 45 degrees f. Frequent turning and positioning

A, C, D, F A. SIADH is the release of ADH despite normal or low plasma osmolality. ADH leads to the reabsorption of water into the circulation. Fluid restriction should result in weight reduction and a gradual rise in serum sodium concentration and osmolality. C. Furosemide (a loop diuretic) may be used to promote diuresis. The serum sodium should be at least 125 mEq/L because it may promote further sodium loss. D. A complication of SIADH is an altered sensorium or seizures. Position the head of the bed flat or elevated no more than 10 degrees to enhance venous return to the heart and increase left arterial filling pressure, thereby reducing the release of ADH. F. Frequent turning, positioning, and range of motion exercises are important to maintain skin integrity and joint mobility.

What clinical manifestations would you expect to find in a patient diagnosed with Irritable Bowel Disease presenting with hypomagnesemia? (Select all that apply) a. Trousseau's sign b. Diminished deep tendon reflexes c. Vertigo d. Muscle cramps e. Urinary retention f. Confusion

A, C, D, F Clinically, hypomagnesemia resembles hypocalcemia. Neuromuscular manifestations are common such as muscle cramps, tremors, hyperactive deep tendon reflexes, Chvostek's sign, and Trousseau's sign. Neurologic manifestations include confusion, vertigo, and seizures.

If a patient is diagnosed with Cardiogenic shock, which drug therapies would the patient be on to treat Cardiogenic shock. Select all that apply: a. Nitrates (nitroglycerin) b. Vasopressors (phenylephrine) c. Diuretics (furosemide) d. Anticholinergics (atropine) e. Inotropes (dobutamine)

A, C, E A. Nitrates (nitroglycerin): decreases preload and afterload, which are responsible for decreasing left ventricular end-diastolic pressure, systemic vascular resistance. Dilates coronary vasculature, dilates arterial and venous beds systemically. C. Diuretics (furosemide): inhibits reabsorption of sodium and chloride at proximal and distal tubule and in the loop of Henle. Unlabeled use: hypertensive emergencies. E. Inotropes (dobutamine): increases contractility, increased cardiac output without marked increases in heart rate by acting on beta-one-receptors in heart; minor alpha and beta-two effects. Unlabeled uses: cardiogenic shock.

The patient is about to receive a transfusion of packed red blood cells. Which of the following observations made by the nurse require intervention prior to beginning the transfusion? (select all that apply) a. The patient has a 24-gauge IV catheter in the left antecubital region b. The unit of packed red blood cells arrived to the unit 15 minutes ago c. The patient's oral temperature is 100˚F (37.8˚C) d. The unit of packed red blood cells is prepared for administration using Y-type tubing and D5W e. The patient's blood type is B+; the blood that arrived from the blood bank is type O-

A, D A. Packed red blood cells can only be administered with a 22-gauge IV catheter or larger. A 24-gauge catheter may be used for other blood products such as platelets. Before beginning any infusion, the patency of the IV needs to be confirmed. D. 0.9% normal saline is the only fluid that should be used with blood product infusions. Using D5W will lead to hemolysis. Y-type tubing should be used.

Which statement(s) would be considered correct in regards to the onset, peak, and duration for the following types of insulin? (Select all that apply). a. Rapid acting insulin has an onset of 10-30 minutes, a peak of 30 minutes - 3 hours, and a duration of 3-5 hours. b. Short acting insulin has an onset of 1.5-4 hours, a peak of 4-12 hours, and a duration of 12-18 hours c. Intermediate insulin has an onset of 30 minutes-1 hour, a peak of 2-5 hours, and a duration of 5-8 hours d. Long acting insulin has an onset of 0.8-4 hours, no pronounced/less defined peak, and a duration of 16-24 hours e. Inhaled insulin has an onset of 12-15 minutes, a peak of 60 minutes, and a duration of 2.5-3 hours.

A, D, E A. Rapid acting insulins, including lispro (Humalog), aspart (NovoLog), and glulisine (Apidra), should be injected within 15 minutes of mealtime. Rapid acting insulin has an onset of 10-30 minutes, a peak of 30 minutes - 3 hours, and a duration of 3-5 hours. D. Long acting insulin has an onset of 0.8-4 hours, no pronounced/less defined peak, and a duration of 16-24 hours. Long acting insulins include glargine (Lantus, Toujeo), detemir (Levemir), and degludec (Tresiba). It is released steadily and continuously and often does not have a peak of action for many people. Glargine and detemir must not be diluted or mixed with any other insulin in the same syringe. E. Inhaled insulin has an onset of 12-15 mins, a peak of 60 minutes, and a duration of 2.5-3 hours. An example of inhaled insulin is Afrezza; it is rapid acting. It is typically administered at the beginning of each meal or within 20 minutes after starting a meal. It must be used in combination with long acting insulin.

A patient diagnosed with chronic kidney disease questions the nurse on his possibility of a kidney transplant. The nurse knows that which of the following factors would cause the patient to be unable to receive a kidney transplant (select all that apply): a. Unmanaged Bipolar II disorder b. Being over age 70 c. Having no familial relatives with a viable kidney to donate d. Disseminated lung cancer e. Being HIV positive f. Chronic respiratory failure

A, D, F A. Contraindications to transplantation include disseminated malignancies, refractory or untreated cardiac disease, chronic respiratory failure, extensive vascular disease, chronic infection, and unresolved psychosocial disorders. D. Contraindications to transplantation include disseminated malignancies, refractory or untreated cardiac disease, chronic respiratory failure, extensive vascular disease, chronic infection, and unresolved psychosocial disorders. F. Contraindications to transplantation include disseminated malignancies, refractory or untreated cardiac disease, chronic respiratory failure, extensive vascular disease, chronic infection, and unresolved psychosocial disorders.

After providing education to a patient with Chronic Kidney Disease (CKD) receiving bi-weekly Peritoneal Dialysis, which of the following statements indicates that dietary coaching has been effective? a. "I can drink as much water as I want as long as my weight and blood pressure are controlled." b. "My daily protein intake should be 2.1 g/kg of my ideal body weight." c. "My nutritional energy sources should come primarily from carbohydrate and fat sources." d. "I will not require supplements to balance my nutritional needs if I'm receiving erythropoietin."

A. "I can drink as much water as I want as long as my weight and blood pressure are controlled." Patients undergoing Peritoneal Dialysis have an unrestricted fluid allowance if weight and blood pressure are controlled.

The nurse is educating a female patient with a recent diagnosis of gastroesophageal reflux disease who is 5'4 and 200 pounds. What statement if made by the patient should communicate to the nurse that the patient may need more education? a. "I will continue my regular nighttime routine of having a glass of wine with a piece of chocolate" b. "I will avoid drinking milk" c. "I will eat small, frequent meals and stop eating 3 hours before laying down." d. "I will start a low-fat diet and weight reduction plan."

A. "I will continue my regular nighttime routine of having a glass of wine with a piece of chocolate". Chocolate and alcohol can decrease lower esophageal sphincter pressure and make patients more inclined to have gastroesophageal reflux disease. If the patient continues their nighttime routine they may still have symptoms of gastroesophageal reflux disease and they will need more education about what the appropriate diet is for this disease process.

A nurse is teaching a patient about prevention of a second UTI. Which of the statements made by the patient requires further intervention? a. "I will stop taking my antibiotic once the symptoms are gone." b. "I should consider drinking unsweetened cranberry juice in my diet." c. "I will urinate regularly approximately every 3-4 hours" d. "I will remember to drink fluids adequately."

A. "I will stop taking my antibiotic once the symptoms are gone." It is important to take all antibiotics as prescribed. Symptoms may improve 1-2 days after therapy, but organisms maybe still present.

A patient diagnosed with an uncomplicated urinary tract infection is given education on how to prevent further infections from occurring. The nurse should intervene if the patient makes which statement? a. "My job is so busy, sometimes I go all day without being able to go to the bathroom." b. "I am sad I can no longer take bubble baths" c. "I will ensure to maintain proper hygiene, especially after a bowel movement." d. "I love cranberry juice! I will buy some on my way home."

A. "My job is so busy, sometimes I go all day without being able to go to the bathroom." Emptying the bladder regularly and completely will help prevent the occurrence of a urinary tract infection. The patient should urinate every 3-4 hours during the day.

A client comes into the clinic complaining of dysuria, frequent urination, urgency, and cloudy urine. For which client would it be expected for the health care provider to order 3 days of trimethropim/sulfamethoxazole (Bactrim)? a. A 22 year old female who tends to drink 4-5 alcoholic beverages on Friday nights. b. A 24 year old male who exercises regularly. c. A 30 year old female with type 2 diabetes. d. A 70 year old male patient who lives in a nursing home and has a foley catheter. e. A 45 year old female who is complaining of costovertebral angle (CVA) tenderness.

A. A 22 year old female who tends to drink 4-5 alcoholic beverages on Friday nights. This is a 22 year old female patient who is presenting signs of a lower urinary tract infection (UTI). It is likely she has uncomplicated cystitis and it can be treated with a 3 day course of antibiotics. Trimethropim/sulfamethoxazole (Bactrim) is a common antibiotic HCP's prescribe for this diagnosis. The fact that she drinks 4-5 drinks on Friday nights is not relevant to this scenario.

What would the nurse assess for in a patient with possible urinary tract calculi? a. A male patient describes pain near the groin area. b. A patient describes the pain on a 0-10 scale as a 2. c. Patient is resting in the chair watching T.V. d. Patient is not nauseous.

A. A male patient describes pain near the groin area. The pain moves around towards the abdomen and down the lower quadrant, men may experience testicular pain while woman complain of labial pain.

During the oliguric phase of Acute Kidney Injury (AKI) which of the following should the nursing consider? a. Electrocardiography (EKG) can show peaked T wave, wide QRS, ST depressed segment b. Patient can be hypovolemic c. Patient can be hypotensive d. During this phase Blood Urea Nitrogen (BUN) and serum creatinine start to stabilize

A. Electrocardiography (EKG) can show peaked T wave, wide QRS, ST depressed segment In a normal functioning kidney, 80 - 90 percent of potassium is excreted. Potassium is a major intracellular cation, with 98 percent of the bodies potassium residing in intracellular fluid. The remaining potassium is in the extra cellular fluid in the intravascular space is approximately 3.5-5.0mEq/L in serum. During the oliguric phase the patient can have an elevated serum potassium. This can be due to several factors: 1. Damaged cells release additional potassium into extracellular fluid especially is AKI is caused by trauma. 2. Blood transfusions cause the release of potassium into extracellular fluid 3. Metabolic acidosis having excess hydrogen ions pushes potassium out of intracellular fluid and it elevates potassium serum. Many patients might be asymptomatic or have increased fatigue with peaked T wave, wide QRS, ST depressed segment. This can be life threatening. Nurse should monitor patient for EKG changes.

A patient is traveling to Malaysia and is at his primary care provider to get vaccinated for the diseases he might be exposed to during his trip. Which vaccine would you expect him to get? a. Hepatitis A vaccine. b. Hepatitis B vaccine. c. Hepatitis C vaccine. d. Hepatitis D vaccine.

A. Hepatitis A vaccine. Hepatitis A is commonly found in developing countries and is spread through the fecal/oral route. There is a vaccine for hepatitis A and it is recommended that people traveling to developing countries get the vaccine.

A patient is diagnosed with Grave's disease. They are scheduled to have a complete thyroidectomy. What post-op complication should the nurse educate the patient on that involves an excessive amount of thyroid hormone being released into circulation and is considered a medical emergency if left untreated? a. Thyroid Storm b. Exophthalmos c. Goiter d. Hyperthyroidism e. Decreased basal metabolic rate

A. Thyroid Storm Thyroid storm also known as thyrotoxic crisis is an acute, severe, and rare condition that occurs when excessive amounts of thyroid hormones are released into circulation. This is a life-threatening emergency if left untreated. Thyroid storm happens as a result of manipulation of the hyperactive thyroid gland which causes an increase in hormones released all at once.

A student nurse is providing health promotion activities to a patient and caregiver to prevent the recurrence of a Urinary Tract Infection. Which statement by the student would require correction from the nurse? a. When providing appropriate hygiene wipe from back to front after urinating b. Maintain adequate fluid intake c. Report to the HCP symptoms or signs of recurrent UTI d. Consider drinking unsweetened cranberry juice or taking cranberry extract tablets 300-400 mg/day

A. When providing appropriate hygiene wipe from back to front after urinating Health promotion activities can help decrease the frequency of infections and provide early detection of infection. Health promotion activities include teaching preventative measures such as 1) emptying the bladder regularly and completely, 2) evacuating the bowel regularly, 3) wiping the perineal area from front to back after urination and defecation, and 4) drinking an adequate amount of liquid each day.

A 65-year-old patient is receiving hemodialysis. They are on Norco for post-surgical pain, and Benazepril for hypertension. The patient starts to unexpectedly bleed out from his surgical incision. What would you do and why? (Select all that apply) a. Give Vitamin K to help the blood clot b. Give protamine sulfate to stem the excessive bleeding c. Stop hemodialysis to focus on the bleeding d. Put pressure on the surgical site to slow the bleeding

B, C B. Protamine sulfate would be used in this situation as the unexpected bleeding is likely caused by the Heparin in the hemodialysis machine. C. Stopping hemodialysis would stop the influx of heparin from the machine. Heparin is used to prevent clotting in the machine.

After receiving report, you decide to go see the patient who was reported to be anxious all night long. Upon entering the room, you take a set of vital signs and discover that their blood pressure is extremely low and their dressings are saturated with blood. You determine that this patient is in hypovolemic shock. What underlying principles do you recognize to pertain to hypovolemic shock? (SELECT ALL THAT APPLY) a. Increasing patient temperature b. Stopping the fluid/blood loss c. Increase circulating blood volume d. Identifying when vital signs began to represent shock e. Treat cause of obstruction f. Obtain cultures

B, C The underlying principles of managing a patient with hypovolemic shock are stopping the loss of fluid and restoring the circulating volume.

A male patient has undergone a hemopoietic stem cell transplant for acute lymphoblastic leukemia (ALL) a couple months ago. From the patient's most recent lab draw, which lab values would be most concerning to the nurse? Select all that applies a. Sodium level of 136 b. Hematocrit of 25% c. Hemoglobin of 8.2 g/dL d. Platelet count of 350,000/μL e. Red blood cell count of 5.0×10^6/μL

B, C This is a pertinent lab to be concerned about because a normal hematocrit level is 39%-50% for males, and this one is on the low end which is also a typical lab value in ALL

A patient undergoing hemodialysis begins to experience complications. Which of the following are known complications associated with hemodialysis? Select all that apply. a. Hypertension b. Muscle cramps c. Loss of blood d. Hepatitis

B, C, D B. Although the cause isn't completely understood, muscle cramps are a known complication of hemodialysis and can be treated by reducing the infusion rate and administering fluids. C. Several circumstances can cause blood loss, including, blood not being completely rinsed from the dialyzer, accidental separation of blood tubing, dialysis membrane rupture, or bleeding after the removal of needles at the end of HD. D. Although the prevalence of Hepatitis B has significantly decreased in patients on hemodialysis, hepatitis c virus is responsible for the majority of cases of hepatitis in dialysis patients. Hepatitis is a known complication of hemodialysis, so there are strict precautions used to protect the patient and staff members.

A 31 year old homeless patient whom has been living under a bridge next to a sewage deposit presents to the emergency department with a temperature of 100.2 degrees and has noticed a yellowish tint to his eyes. The nurse would suspect the patient has contracted what hepatic condition? a. Hepatitis C b. Hepatitis A c. Ascites d. Jaundice

B. Hepatitis A Hepatitis A can be contracted from crowded conditions, poor personal hygiene, sexual contact, IV drug users, poor sanitation, contaminated food and water (like a sewage deposit). Infection presents with mild flu-like illness and jaundice which is a yellowish discoloration of body tissues which can be seen in the eyes sometimes.

When providing education to a group of adults targeted by a community outreach program, the nurse should include the following information to teach ways to decrease the risk of contracting HIV. (Select all that apply) a. Kiss individuals only after knowing their HIV status b. Increase safe sex practices, including always wearing a condom and decrease number of sex partners c. Don't share drug equipment including needles, cookers, cotton, rinse water, straws, or pipes d. Safe sexual activities include "hand jobs" and other mutual masturbation techniques

B, C, D Safe sexual activities eliminate the risk of exposure to HIV in semen and vaginal secretions. Abstaining from all sexual activities is an effective way to accomplish this goal, but there are safe options for those who cannot or do no wish to abstain. Limiting sexual activities in which the mouth, penis, vagina, or rectum does not encounter a partner's mouth, penis, vagina, or rectum eliminates contact with blood semen, or vaginal secretions. Safe activities include masturbation "hand jobs" and other "no contact" requirements. The safest method is to abstain from drugs, but this may not be a viable option for users who are not prepared to quit. The risk of HIV can be eliminated if they do not share equipment. Including injecting equipment (needles, syringes), cookers (spoons or bottle caps to mix the drug), cotton, and rinse water. Equipment used to snort (straws) or smoke (pipes) drugs can also be contaminated with blood and should not be shared.

The nurse is providing patient and caregiver teaching for a patient recently diagnosed with Neutropenia. The nurse will include which of the following in his/her teaching: (Select all that applies) a. The patient should not use antibacterial hand gel to sanitize their hands. b. The patient should notify the health care provider if they have a fever of 100.4 or above. c. The patient should do moderate exercises and walking. d. The patient should avoid activities that involve a large number of people. e. The patient should report any changes in urination and bowel habits.

B, C, D, E B. Early identification of an organism that is potentially infective is essential. Serial blood cultures from a peripheral site and a venous access device should be collected to identify the organism. Broad-spectrum antibiotics should be administered within an hour until a causative organism is found. C. Keeping up on daily exercises is important for the physical and mental health of the patient. Coaching on safe activities can reduce social isolation risks and improve quality of life. The patient can maintain activities and exercises as long as they are avoiding crowds. D. Large crowds increase the risk of transmission of an infectious organism. The patient with neutropenia has a decrease in their total white blood cell count making them predisposed to an infection from opportunistic pathogens as well as nonpathogenic organisms. Air, food, water, and equipment provide additional opportunities for the transmission of an infection. Avoiding large crowds reduces these risks. E. Clinical manifestations related to an infection in a patient who has neutropenia include diarrhea, rectal tenderness, vaginal itching, and discharge. Other common clinical manifestations related to an infection are sore throat, dysphagia, ulcerative lesions, buccal mucosa, shortness of breath, and a nonproductive cough.

A 42-year-old female has been diagnosed with hyperthyroidism due to Grave's disease. A student nurse is educating the patient about the disease, its sign and symptoms and precipitating factors. The registered nurse is confident that student nurse understands this illness when the student nurse makes which following statements? (SELECT ALL THAT APPLY) a. Graves disease is caused by a virus and it affects your thyroid levels but with the proper medication, you can cure this infection. b. One hallmark sign for Graves' disease is exophthalmos c. Clinical manifestations of this illness include warm, smooth, and moist skin d. Clinical manifestations of this illness include thick, cold, and dry skin. e. Without proper treatment this disease can result in hypothyroidism.

B, C, E B. Clinical manifestations of hyperthyroidism can be directly related to the effect of excess circulating thyroid hormones. A classic finding in Graves' Disease is exophthalmos, which is a protrusion of the eyeballs from the orbits that are usually bilateral. It happens from increased fat deposits and fluid, basically edema, in the orbital tissues and ocular muscles. Because of the increased pressure, the eyeballs are forced outward. This can lead the eyes to become dry and irritated. C. Patients with Graves' Disease have hyperthyroidism. Clinical manifestations of hyperthyroidism include warm smooth and moist skin when referring to the integumentary system. Patients often have a heat intolerance. The reason that patients have a sensitivity to heat and have moist skin is because the thyroid gland has lost the ability to respond to the normal control of the pituitary gland which regulates temperature. E. With Graves' Disease, there is often remissions and exacerbations with or without treatment. Because of these continuous exacerbations, it may lead to tissue destruction of the thyroid which can ultimately put the patient into hypothyroidism. Hypothyroidism is caused by destruction of the thyroid tissue, or defective hormone synthesis. So you can see that if a patient is continually in this remission and exacerbations from Graves' Disease, that can cause damage to their thyroid tissue thus leading to hypothyroidism at some point.

The nurse is doing discharge teaching for a patient with a new diagnosis of thrombocytopenia. Which statement, if made by the patient, would indicate that the patient understands the teaching? (SELECT ALL THAT APPLY) a. "I can't wait to get back to my weights class at the gym!" b. "I will talk with my Dr. before my pedicure appointment next week." c. "I will stop taking my herbal supplements and over the counter medications until I speak with my Dr." d. "I will replace my toothbrush with one with medium bristles." e. "I will contact my Dr. if I get a headache, or my vision changes."

B, C, E B. Talk with your HCP before you have any invasive procedures done, such as pedicures, manicures, or dental cleanings. C. Any medications or herbal supplements that prolong bleeding, such as aspirin, should be avoided. E. Headache and/or vision changes are a manifestation of bleeding.

34-year-old male, Jeffery Young, is on his way to the Emergency Department following a bar fight. Emergency Medical Technicians have reported to dispatch an estimated blood loss of 2000 ml. With this information the Nurse knows to expect which manifestations of acute blood loss? (Select all that apply) a. Shock b. Low Blood Pressure c. Rapid, thready pulse d. Lactic acidosis e. Cold, clammy skin

B, C, E Low blood pressure, low central venous pressure, and cardiac output below normal at rest; air hunger; rapid, thread pulse and cold, clammy skin. These are manifestations of acute blood loss for a volume loss of 2000 ml

When providing education to a patient newly diagnosed with cirrhosis, what information should be included? (select all that apply) a. Inform the patient that cirrhosis is an acute condition that can be alleviated with the correct drug regimen b. Avoid aspirin and NSAIDs to prevent hemorrhage c. Limit alcohol consumption to one drink per week d. Teach symptoms of complications and when to seek medical attention e. Avoid hepatotoxic over-the-counter (OTC) drugs

B, D, E B. In patients with cirrhosis, especially if they have esophageal or gastric varices as a comorbidity, use of aspirin and NSAIDs is discouraged. The damage cirrhosis causes to the liver lowers its ability to assist in clotting and taking aspirin and NSAIDs, which also have anticoagulant properties, can increase the risk of hemorrhage D. Patient should be informed of signs and symptoms to look for that may indicate worsening of their condition. These can include peripheral edema (swelling in extremities), signs of bleeding, ascites (build-up of fluid in peritoneal cavity), alterations in mental status, asterixis (flapping tremors), etc. If any of these symptoms occur, the patient should seek medical attention right away E. OTC drugs that may be toxic to the liver (ex: acetaminophen in high doses) should be avoided because the damage that cirrhosis causes to the liver makes it unable to metabolize those drugs

A patient has been diagnosed with gastroesophageal reflux disease (GERD). What should the nurse include in the education for managing GERD? a. "You can eat whatever you like since what you eat doesn't affect GERD." b. "You should not lay flat for at least 2-3 hours after eating." c. "Eat a late-night snack, it will buffer the acid in your stomach decreasing irritation." d. "Take your PPI medication before you eat breakfast." e. "Decreasing life stressors will help decrease your symptoms."

B, D, E B. The patient should not be supine for 2 to 3 hours after a meal. D. Therapy should start with once a day dosing, before the first meal of the day. E. If stress seems to cause symptoms, discuss measures to cope with stress.

A nurse is educating a female patient about to be discharged from the hospital after being treated for a urinary tract infection (UTI). The nurse recognizes the patient correctly understands the education material when they make the following statements, EXCEPT? (SELECT ALL THAT APPLY) a. I should urinate approximately every 3-4 hours throughout the day b. I should take bubble bathes as a way to get everything clean down there c. I should be emptying my bladder before and after sexual intercourse d. I should be wiping from front to back after urinating e. I should stop taking my antibiotics once the symptoms of my urinary tract infection (UTI) are gone

B, E B. Avoid vaginal douches and harsh soaps, bubble baths, powders, and sprays in the perineal area. E. Emphasize the importance of taking the full course of antibiotics. Often patients stop antibiotic therapy once symptoms disappear. This can lead to inadequate treatment and recurrence of infection or bacterial resistance to antibiotics

A nurse is teaching a patient newly diagnosed with Interstitial cystitis (IS) and Painful Bladder Syndrome (PBS) about his condition. The nurse realizes that further teaching is necessary when the patient states? a. "I may experience pain and bothersome lower urinary tract symptoms (LUTS) such as frequency and urgency". b. "I will be taking antibiotics to treat interstitial cystitis". c. "If my condition becomes severs, I may urinate as often as 60 times per day". d. "Pain is often temporarily relieved by urination".

B. "I will be taking antibiotics to treat interstitial cystitis". Urine cultures do not find any bacteria or other organisms in the urine. Furthermore, people with IC/PBS do not respond to antibiotic therapy.

The nurse knows, which of the following patients is most at risk of having another flair up of their chronic pancreatitis? a. A 72-year-old female, who enjoys eating fried foods regularly and states she prefers to eat late dinners just before bedtime. b. A 23-year-old male, who was hospitalized with spleen injury after a MVA, who states he drinks several times a week with his friends, and takes NSAIDs to treat abdominal pain from his accident. c. A 56-year-old male, who prefers drinking milk instead of water with meals, and states his diet consists of mostly legumes and fish. d. A 28-year-old female, with poly cystic ovarian syndrome and a BMI of 31, who states she binges on sweets when she is feeling stressed from work and school.

B. A 23-year-old male, who was hospitalized with spleen injury after a MVA, who states he drinks several times a week with his friends, and takes NSAIDs to treat abdominal pain from his accident. This patient is most at risk for aggravating his chronic pancreatitis he drinks heavily and sustained abdominal trauma recently which puts him at higher risk. Encourage the patient to eliminate alcohol intake, especially if he or she has had any previous episodes of pancreatitis. Recurrent attacks of pancreatitis may become milder or disappear with the discontinuance of alcohol use.

A student nurse is discussing risk factors for renal cell carcinoma with an experienced nurse. The experienced nurse should intervene if the student describes which of the following as a risk factor for developing this type of cancer? a. Smoking one pack of cigarettes per day b. A blood pressure of 110/76 c. A body mass index of 31 kg/m2 d. Workplace exposure to asbestos and gasoline

B. A blood pressure of 110/76 Risk factors include obesity; hypertension; and exposure to asbestos, cadmium, and gasoline. A blood pressure of 110/76 is within normal limits and does not indicate hypertension.

Which patient is at most risk for developing Chronic Kidney Disease (CKD)? a. A 54-year-old white male with uncontrolled hypertension. b. An 80-year-old Native American with diabetes and polypharmacy. c. A 64-year-old African American female with obesity and a Hgb A1-c of 6. d. A 60-year-old Hispanic female with hyperlipidemia and blood pressure of 130/80.

B. An 80-year-old Native American with diabetes and polypharmacy. Native Americans have a rate of CKD twice that of whites. The rate of CKD is six times higher among Native Americans with diabetes than among other ethnic groups with diabetes. The aging kidney and polypharmacy are precursors to developing Acute Kidney Injury (AKI). However, when a patient does not recover from AKI, then CKD may develop.

Lifestyle modifications are crucial for the management and prevention of gastroesophageal reflux disease (GERD). Which of the following is a strategy for good management to teach a patient who was recently diagnosed? a. Eat a small meal close to bedtime to ease the onset of GERD b. Avoid alcohol and caffeinated beverages c. Avoid taking sucralfate (Carafate) d. If a PPI is being taken, it is best to take it right before bedtime

B. Avoid alcohol and caffeinated beverages Caffeinated beverages, as well as alcohol, cause an almost immediate, marked decrease in lower esophageal sphincter pressure. Because of this, alcohol and caffeinated beverages should be avoided.

A student nurse is explaining the probable sites for an arteriovenous fistula to the patient. Which statement does the preceptor know suggests understanding of placement on the part of the student nurse? a. Brachial and antecubital area b. Cephalic or basilic vein with radial artery c. Femoral vein d. Internal jugular

B. Cephalic or basilic vein with radial artery The brachial or basilic vein with the radial artery are used with an arteriovenous fistula

A 45-year-old patient comes into the ER with peripheral edema, massive proteinuria, hypertension, and foamy urine. The nurse suspects nephrotic syndrome. What labs does the nurse expect to see? A. Increased serum albumin B. Decreased total serum protein C. Decreased serum cholesterol D. Increased serum calcium

B. Decreased total serum protein Characteristic laboratory findings include decreased serum albumin, decreased total serum protein, and elevated serum cholesterol.

A 12-year-old patient presents with poor skin turgor, dry mucus membranes, tachycardia, orthostatic hypotension, lethargy, weakness, and has rapid deep respirations with a sweet fruity odor. The nurse suspects this patient is suffering from: a. Hyperosmolar hyperglycemic syndrome b. Diabetic Ketoacidosis c. Insulin excess d. Dehydration

B. Diabetic Ketoacidosis Diabetic Ketoacidosis is seen mostly in type 1 diabetes. It is a profound deficiency in insulin is characterized by hyperglycemia, ketosis, acidosis, and dehydration. The body has begun to break down fats and proteins for energy from the lack for glucose entering the cell. This caused all the above signs and symptoms. Acidosis causes Kussmaul respirations too in an attempt to compensate for acidotic state. Increased acetone can cause a sweet fruity odor on the breath

You are planning discharge education for a patient who was diagnosed with acute diverticulitis. Included in the patients dietary education would be: a. Eat a diet high in carbohydrates and avoid nuts and seeds. b. Eat a diet high in fiber, mainly from fruits and vegetables. c. Increase the amount of red meat and fat in diet to promote protein absorption. d. Diet modifications are not necessary. Continue with your current diet, but add a dietary fiber supplement and stool softeners to your daily routine.

B. Eat a diet high in fiber, mainly from fruits and vegetables. It is recommended that the patient has a diet high in fiber and most of that fiber coming from fruits and vegetables. Decreasing the level of red meat and fat are also recommended to prevent diverticulitis.

When educating a patient newly diagnosed with type 2 diabetes you interfere if the patient makes which INCORRECT statement? a. Being obese or overweight is a risk factor for developing type 2 diabetes. b. In type 2 diabetes the pancreas produces sufficient amounts of insulin. c. A family history of type 2 diabetes may be a risk factor d. This disease is seen less frequently in children.

B. In type 2 diabetes the pancreas produces sufficient amounts of insulin. In type 2 Diabetes, the pancreas produces some insulin. However, the body not produce enough insulin, or does not use it effectively, or both.

The nurse is providing care to a patient diagnosed with a proximal small intestine obstruction. The nurse is monitoring for common manifestations of a proximal small intestine obstruction and knows they include all of the following except: a. Frequent and copious vomiting b. Increased abdominal distention c. Frequent cramping d. Rapid onset of pain e. Rapid dehydration

B. Increased abdominal distention The signs of colonic obstruction include abdominal distention, either absolute constipation or a marked change in bowel function, and lack of flatus. Vomiting is rare.

A client diagnosed with chronic kidney has the following ABG results; pH 7.26, PaCO2 30 mm Hg, HCO3 16 mEq/L, PaO2 90 mm Hg, and O2 saturation of 96%. As the nurse, you will interpret these results as a. Respiratory Acidosis b. Metabolic Acidosis c. Respiratory Alkalosis d. Metabolic Alkalosis

B. Metabolic Acidosis Metabolic Acidosis occurs when an acid other than carbonic acid accumulates in the body or when bicarbonate is lost from body fluids. The compensatory response to metabolic acidosis is to increase CO2 excretion by the lungs. The patient often develops kussmaul respiration, and in addition, the kidneys attempt to excrete addition acid.

Stacey, a 61-year-old woman, states that almost every time she sneezes, coughs or laughs she urinates a little bit and is unable to control it from happening. What type of urinary incontinence is this patient most likely experiencing? a. Urge Incontinence b. Stress Incontinence c. Overflow Incontinence d. Functional Incontinence

B. Stress Incontinence Stress incontinence is a sudden increase in intraabdominal pressures causing an involuntary passage of urine. This can occur from coughing, laughing, sneezing or physical activities like heavy lifting. Leakage is usually in smaller amounts and may or may not be a daily occurrence.

The nurse is explaining the goals of antiretroviral therapy to a patient. Which explanation by the nurse is correct on how this therapy will slow the progression of HIV? a. The goal of antiretroviral therapy is to cure HIV by eliminating the viral load from the blood. b. The goal of antiretroviral therapy is to decrease the amount of virus load in the blood and to keep the CD4+ T cell count at 800 to 1200 cells/µL. c. The goal of antiretroviral therapy is to maintain the CD4+ T cell count at 200 cells/µL. d. The goal of antiretroviral therapy is to reduce the risk of lipodystrophy, a common metabolic disorder found in patients with HIV.

B. The goal of antiretroviral therapy is to decrease the amount of virus load in the blood and to keep the CD4+ T cell count at 800 to 1200 cells/µL. The goal of antiretroviral therapy is to keep CD4+ T cell counts high. Immune problems begin to occur when this number drops below 500 cells/µL. The normal range for CD4+ T cells is 800-1200 cells/µL These cells are lymphocytes that help fight off infections and disease. With the help of antiretroviral therapy, the replication of the HIV virus can be significantly slowed, and the body can produce more CD4+ T cells to replace the destroyed cells.

A nurse is administering parenteral nutrition to their patient. What action by the nurse would require the charge nurse to intervene? a. The nurse verifies the parenteral nutrition rate and dose before administration. b. The nurse realizes the feeding is behind schedule and speeds it up. c. The feeding finishes sooner than expected so the nurse starts a 10% dextrose solution. d. The nurse attaches feeding tubing to the patient's peripherally inserted central catheter.

B. The nurse realizes the feeding is behind schedule and speeds it up. The nurse would not speed up the feeding even though it was behind. This could cause an electrolyte imbalance in the patient or cause refeeding syndrome. The proper action would be to correct the speed of the feeding to what it should be and ensure that it is feeding the proper rate after the change.

The nurse is caring for a patient who has GERD. The nurse determines that further education is necessary when the family makes which statement? a. Our dad should avoid fried and fatty foods. b. We should include plenty of fruit and juices into his new diet. c. Losing weight and wearing loose clothing will help our dad. d. Raising the head of his bed will help manage our dad's GERD.

B. We should include plenty of fruit and juices into his new diet. Citrus and other juice drinks tend to be very acidic and may cause further acid reflux.

When assessing a patient with the diagnosis of Grave's disease, the nurse anticipates the following manifestations EXCEPT: a. exophthalmos b. thick, brittle nails c. laboratory results showing low thyroid stimulating hormone (TSH) and elevated thyroxine (T 4 ) d. intolerance to heat

B. thick, brittle nails Thin, brittle nails detached from nail bed (onycholysis) are a symptom of hyperthyroidism whereas thick, brittle nails are found in hypofunction of the thyroid.

A patient has recently been diagnosed with cancer of the bladder. She has become incontinent and is undergoing urinary diversion surgery tomorrow. At the preoperative appointment you are educating the patient on what to expect postoperatively. You know the patient requires further teaching if they make which statement? a. It is normal for my stoma to shrink within the first couple weeks of surgery. b. I will need to limit my fluid intake at home to prevent excess urine from seeping from my stoma. c. I will be fitted for a permanent appliance 7-10 days after my surgery. d. After my surgery I will need to continue using my incentive spirometer to prevent my lungs from collapsing.

B: "I will need to limit my fluid intake at home to prevent excess urine from seeping from my stoma" You would stop the patient if they are making a statement about limiting their fluid intake upon returning home. It is important for the patient to maintain consistent urine output. The patient is encouraged to have a high fluid intake in order to "flush" the ileal conduit. The urine will not seep from the stoma.

A 43 year old woman is diagnosed with gastroesophageal reflux disease (GERD). What guidelines can the nurse provide to her for managing her symptoms? SELECT ALL THAT APPLY. a. Lie down after eating to relieve pressure b. Having peppermint or tea after eating can soothe any discomfort c. Avoid late night snacking d. Eating small, more frequent meals is better than eating a few large meals per day e. Drinking fluids between meals and not with meals can help prevent reflux

C, D, E C. Eating before bedtime should be avoided to prevent food from traveling back up the esophagus. It is recommended that patients with GERD avoid eating within 3 hours of bedtime. D. Eating small, frequent meals can help prevent over distention of the stomach. This reduces symptoms of GERD. E. Drinking fluids between meals, but not with meals will help prevent over distention of the stomach. This reduces symptoms of GERD

Select all that apply: You have a patient who has been diagnosed with sepsis. What would be appropriate patient and family education in regards to this diagnosis? a. Sepsis is a low grade infection which responds quickly to antibiotics, and is overall a short term illness b. Initial treatment will begin with narrow spectrum antibiotics as quickly as possible to combat the spreading infection c. Sepsis is a serious illness that can cause organ damage and will require aggressive treatment with antibiotics, intravenous fluids and a high level of care d. Initial treatment will include fluid replacement, oxygen therapy, and broad spectrum antibiotics e. Sepsis if left untreated can progress to septic shock which can cause organ failure and death

C, D, E C. Sepsis and septic shock have a high incidence worldwide with a mortality rate of 25% or higher. Making this a very serious illness requiring aggressive interventions including antibiotics, IV fluids, and oxygen. D. Initial treatment for sepsis will include rapid fluid replacement, supplemental oxygen, and broad-spectrum antibiotics. E. Sepsis and septic shock have a high incidence of mortality causing irreversible organ damage, coma and death.

Adriana Ottwell is in the observation unit following a cholecystectomy. The nurse receives orders from the physician to prepare the patient for discharge. Patient education is successful if Adriana makes what statement concerning ambulatory care? a. "I can stop taking my anti-biotics as soon as the surgical wound heals." b. "Vomiting is normal after my surgery and I shouldn't be concerned." c. "I should take it easy but I can return to normal activities gradually." d. "I can pick up my 18 month old child when he wants to be held."

C. "I should take it easy but I can return to normal activities gradually." Early ambulation speeds recovery, but normal activities are resumed gradually, with planned rest periods.

A patient's arterial blood gas (ABG) results are: PH; 7.31, PaCO2; 54 mmHg, HCO3; 25 mEq/L. What kind of acid-base imbalance is this? a. Metabolic acidosis, uncompensated b. Respiratory alkalosis, compensated c. Respiratory acidosis, uncompensated d. Metabolic alkalosis, uncompensated e. Respiratory acidosis, compensated

C. . Respiratory acidosis, uncompensated Respiratory acidosis occurs whenever the person hypoventilates. [This] leads to a buildup of CO2, resulting in an accumulation of carbonic acid in the blood. Renal compensatory mechanisms begin to operate within 24 hours. The kidneys conserve HCO3 (bicarbonate) and secrete increased concentrations of H+ (hydrogen) into the urine. Until the renal mechanisms have an effect, the serum HCO3 level will usually be normal, and then it will increase.

A patient is diagnosed with urge incontinence. The nurse knows that which condition/circumstance could lead to the development of urge incontinence? a. Prostate surgery for BPH or prostate cancer b. Diabetic neuropathy c. Alzheimer's disease d. Spinal cord lesion above S2

C. Alzheimer's disease Urge incontinence is often referred to as an "overactive" bladder. It is characterized by a feeling of extreme urinary urgency. It can be caused by nervous system disorders such as stroke, Alzheimer's disease, brain tumor, or Parkinson's disease.

When providing education to a patient recently diagnosed with Sickle Cell Disease, what should the nurse instruct the patient about managing symptoms and preventing a crisis? a. Take weekly high-intensity, high-elevation hikes to ensure cardiac health b. Drink 1L/day of water c. Begin screening for retinopathy at the age of 10 d. Seek medical attention two weeks after the onset of infection if condition worsens

C. Begin screening for retinopathy at the age of 10 Teach patients with SCD to avoid high altitudes, maintain adequate fluid intake, and treat infections promptly. Screening for retinopathy should begin at age 10.

A patient asks the student nurse to explain common symptoms of a lower urinary tract infection. The student nurse would be correct if they gave all the symptoms except which of the following? a. Dysuria b. Suprapubic discomfort c. Flank pain d. Urgency

C. Flank pain Flank pain, chills, and fever indicate an infection involving the upper urinary tract (pyelonephritis).

The nurse would question the order for enteral nutrition in patients with which of the following diagnosis? a. Anorexia b. Chemotherapy patient c. Gastrointestinal Obstruction d. Extensive burns

C. Gastrointestinal Obstruction Contraindications for enteral therapy include those with a malfunctioning GI system.

A patient is presenting to the emergency department with confirmed anaphylactic shock after eating peanuts. Which order by the doctor would the nurse question in regard to the treatment of this patient? a. Epinephrine IM b. Albuterol c. Nitroglycerin d. Diphenhydramine

C. Nitroglycerin Hypotension results from leakage of fluid out of the intravascular space into the interstitial space as a result of increased vascular permeability and vasodilation. Vasodilators, such as nitroglycerin, would be contraindicated

After a patient comes into the hospital for an AKI and you notice that he/she is experiencing severe dehydration, and oliguria. The patient does not encompass any damage to the kidney tissue but is experiencing decreased circulating blood volume, which stage of AKI would this patient be in? a. Acute tubular necrosis b. Intrarenal c. Prerenal d. Postrenal

C. Prerenal The patient would be in the prerenal stage because this stage of an acute kidney injury has oliguria but does not have any damage to the kidney tissue and can be reversed with adequate treatment. The oliguria in this stage can be caused by severe dehydration, heart failure, and decreased cardiac output.

A nurse is caring for a patient recovering a motor vehicle accident (MVA) is experiencing Neurogenic Shock, which action by the nurse should be questioned? a. The use of phenylephrine b. The use of atropine c. Rapid infusion of IV fluids d. The use of a collar to promote spinal stability

C. Rapid infusion of IV fluids Infuse fluids cautiously as the cause of hypotension is not related to fluid loss.

A 29-year-old anorexic female patient recently started receiving parenteral nutrition via central catheterization. During your re-assessment you notice the patient has developed some general edema, and tells you she is having trouble breathing, feels like her heart is "skipping beats," and is concerned about her lower extremities feeling tingly. What is most likely occurring with your patient? a. Hyperventilation brought on by anxiety related to new procedure b. Relative hypovolemic shock related to systemic vasodilation c. Refeeding syndrome related to electrolyte imbalances and fluid retention d. Hyperphosphatemia related to malnutrition and fluid overload

C. Refeeding syndrome related to electrolyte imbalances and fluid retention The hallmark presenting signs and symptoms of refeeding syndrome are fluid retention and electrolyte imbalances. This metabolic problem is a complication of parenteral nutrition that often occurs in malnourished patients starting aggressive nutritional therapy.

Which of the following is true of hypovolemic shock? a. blood products are not initiated as volume replacement until volume loss is greater than 40% b. the size of the vascular compartment increases while the fluid volume decreases c. relative hypovolemia is seen in burn patients d. drug therapy for hypovolemic shock is administration of vasopressors and antibiotics

C. Relative hypovolemia is seen in burn patients Due to increased capillary permeability, there is leakage of fluid from the vascular space to the interstitial space, resulting in relative hypovolemia.

A patient arrives to the Emergency Department with a small foul looking laceration on his right heel, a blood pressure of 98/52, a respiratory rate of 24, and a heart rate of 110. After receiving a 500ml bolus of Normal Saline the patient's blood pressure is 96/48, he has a respiratory rate of 28, a heart rate of 130 and his lab results are showing a decrease in kidney and liver function from baseline. What do you expect this patient to be diagnosed with? a. Sepsis b. Hypovolemic Shock c. Septic Shock d. Inadequate tissue perfusion related to sepsis

C. Septic Shock Septic shock is a subset of sepsis characterized by persistent hypotension despite adequate fluid resuscitation requiring vasopressors, along with inadequate tissue perfusion resulting in tissue hypoxia

A patient has been receiving chemotherapy and radiation for a diagnosis of non-Hodgkin's lymphoma. In your morning assessment, you notice the patient has new onset facial edema, distention of head and neck veins, and dyspnea. Which oncologic emergency would be suspected? a. Hypercalcemia b. Spinal Cord Compression c. Superior Vena Cava Syndrome (SVCS) d. Tumor Lysis Syndrome (TLS)

C. Superior Vena Cava Syndrome (SVCS) The manifestations in the patient are all seen in SVCS in addition to headache, seizures, and often a mediastinal mass found on a chest x-ray. This complication is common with lung cancer, metastatic breast cancer, and non-Hodgkin's lymphoma

A student nurse is explaining to a patient about the health risks associated with gynoid obesity. Which statement, if made by the student nurse would require the primary nurse to intervene? a. Telling the patient they are at risk for developing osteoporosis b. Telling the patient they are at an increased risk for developing varicose veins c. Telling the patient they are at risk for developing diabetes mellitus d. Telling the patient that fatty acids are stored as triglycerides

C. Telling the patient they are at risk for developing diabetes mellitus. Diabetes mellitus is a health risk for patients with android obesity, where the fat primarily located in the abdominal area.

The nurse is taking care of a patient who just arrived into the ED with suspected sepsis. The nurse would know one of the clinical manifestations of septic shock are? a. Hypoglycemia b. Hyperthermia c. Vasodilation d. Bradypnea

C. Vasodilation Septic shock has three major pathophysiologic effects: vasodilation, maldistribution of blood, and myocardial depression. Patients may be euvolemic, but because if acute vasodilation, relative hypovolemia and hypotension occur.

The nurse is caring for a male patient who is suspected of being in metabolic syndrome. The nurse understands that this puts the patient at greater risk for cardiovascular disease, stroke, and diabetes mellitus. When the nurse inspects the patient's chart, he would expect to see which of the following? a. Waist circumference of 42 in, triglyceride level of 136 mg/dL, fasting blood glucose of 120 mg/dL, and a blood pressure of 122/78. b. Waist circumference of 38 in, triglyceride level of 144 mg/dL, fasting blood glucose of 98mg/dL, and a blood pressure of 125/66. c. Waist circumference of 44 in, triglyceride level of 164 mg/dL, HDL level of 52 mg/dL, and a medication order for hydrochlorothiazide 12.5 mg PO BID. d. Waist circumference of 32 in, triglyceride level of 143 mg/dL, a medication order for atorvastatin 10mg PO QD, and blood pressure of 133/72.

C. Waist circumference of 44 in, triglyceride level of 164 mg/dL, HDL level of 52 mg/dL, and a medication order for hydrochlorothiazide 12.5 mg BID. The signs of metabolic syndrome are impaired fasting blood glucose, hypertension, abnormal cholesterol levels, and obesity. Patient's with a waist circumference of >40 in, triglyceride levels of >150mg/dL, a fasting blood glucose of >110mg/dL, an HDL level of <40 mg/dL, a systolic BP >130 mmHg or diastolic BP >85 mmHg, or treatment for one of the above are all criteria for metabolic syndrome. The patient must meet 3 of the 5 above listed criteria in order to be considered to be in metabolic syndrome. This patient presents with a waist circumference of 44 in., an elevated triglyceride level of 164 mg/dL and is being treated for hypertension with hydrochlorothiazide 2.5 mg PO BID. This patient's HDL is not decreased, but this patient does meet three criteria for metabolic syndrome.

A patient is admitted to the emergency department with signs and symptoms of shock. Your priority is in assessing airway, breathing, circulation, and monitoring vitals. His blood oxygen saturation (SpO2) is at 86% on room air, blood pressure 89/68, heart rate 130, respirations 22, and temperature 98.9 degrees Fahrenheit. After taking vitals, you continue to monitor your patient for which signs and symptoms that indicate shock progression? Select all that applies a. Polyuria b. Warm, flushed skin c. Hyperreactive reflexes d. Anuria e. Hypothermia

D, E Classic manifestations of stages of shock include anuria, cold, clammy, mottled skin, hypoactive reflexes or areflexia (loss of reflexes), and hypothermia.

A female patient with an uncomplicated UTI requires further teaching by the nurse when they make which statement? a. "I should finish my dose of Bactrim that the doctor prescribed for me in three days unless otherwise noted." b. "I should drink at least six 8-oz glasses of water every day to prevent a further UTI from occurring." c. "I should urinate regularly (approximately every 3-4 hours) instead of holding it in." d. "I should take regular bubble baths in order to properly cleanse the urethral area."

D. "I should take regular bubble baths in order to properly cleanse the urethral area." All vaginal douches, harsh soaps, bubble baths, powders, and sprays in the perineal area should be avoided to prevent irritation of the urethra and any possibility of bacteria. Proper hygiene of the female genital area includes separating the labia when cleaning the perineal region, wiping from front to back after urination, and cleansing with warm, soapy water after each bowel movement.

After providing education to a patient with gastroesophageal reflux disease, the nurse knows that more education is necessary when the patient makes which statement? a. "I will take omeprazole every morning on an empty stomach" b. "I will be sad to say goodbye to my bedtime chocolate milk" c. "I will start eating smaller, more frequent meals" d. "I will be sure to sleep without a pillow so that the contents of my stomach are more evenly distributed"

D. "I will be sure to sleep without a pillow so that the contents of my stomach are more evenly distributed" Lying flat will likely aggravate the patient's symptoms. The patient should be taught to sleep with the head elevated to promote esophageal emptying.

A 26 year old male presents to the Emergency Department with 2 days of bloody stools, abdominal cramping, and watery diarrhea. He is diagnosed with E. coli poisoning. When explaining what he ate the past few days, which items might you suspect as the E. coli carrying culprit? a. A bowl of oatmeal with brown sugar and raisins b. Vegetable soup c. A chocolate brownie d. A cheeseburger and fries

D. A cheeseburger and fries Contaminated beef, pork, milk, cheese, and cookie dough are the most common sources of E. coli food poisoning

What is Inflammatory bowel disease (IBD)? a. Inflammation of the mucosa of the stomach and small intestine with sudden diarrhea, nausea, vomiting, fever, and abdominal cramping. b. Disorder characterized by chronic abdominal pain or discomfort and alteration of bowel patterns between diarrhea and constipation. c. Chronic symptom of mucosal damage caused by reflux of stomach acid into the lower esophagus. d. Chronic inflammation of the GI tract characterized by periods of remission interspersed with periods of exacerbation. e. Syndrome defined by difficult or infrequent stools; hard, dry stools that are difficult to pass; or a feeling of incomplete evacuation.

D. Chronic inflammation of the GI tract characterized by periods of remission interspersed with periods of exacerbation. IBD is a chronic inflammation of the GI tract characterized by periods of remission interspersed with periods of exacerbation

Central Venous Access Devices (CVADs) have several potential complications. Careful monitoring and assessment may assist in early identification of potential complications. What is a manifestation of a pneumothorax, if it would occur? a. Sluggish infusion or aspiration b. Dysrhythmias c. Systemic: fever, chills, malaise d. Decreased or absent breath sounds

D. Decreased or absent breath sounds Decreased or absent breath sounds is a manifestation of a pneumothorax.

When educating your patient about the differences between glomerulonephritis and nephrotic syndrome, which from the following choices would the nurse INCLUDE in their education. a. Glomerulonephritis is defined as the glomerulus becoming excessively permeable to plasma proteins. b. Nephrotic syndrome is defined as the inflammation of the glomeruli, affecting both kidneys equally. c. Nephrotic syndrome can be caused by streptococcal infections d. Glomerulonephritis can either be acute or chronic. e. Edema, a manifestation of nephrotic syndrome, will show around the eye at first (peri-orbital), and then progress to involve the whole body.

D. Glomerulonephritis can either be acute or chronic. Glomerulonephritis can be classified as either acute or chronic with onset of symptoms of acute being rapid and possibly reversible, therefore this answer is correct.

A patient is coming back post operatively from a thyroidectomy. The nurse should take all of the following actions EXCEPT: a. Position patient in Semi-Fowlers position b. Asses for hemorrhage every 2 hours c. Head support to avoid stress on the sutures d. Have patient preform range of motion and neck exercises immediately

D. Have patient preform range of motion and neck exercises immediately. This answer is incorrect because you want to avoid movement of the neck to prevent pulling of the suture wounds. You are placing the patient in semi fowlers, avoiding movement and placing pillows around the patient to prevent any movement so you do not want to add strain to the neck by having the patient perform any time of exercise.

A patient is being seen for a diabetes follow up appointment with their healthcare provider. While waiting for the doctor to arrive, the patient starts to share about their diabetes management. The patient states that they will ask the health care provider to increase their dose of insulin because they have been waking up with elevated levels of glucose despite their adherence to their diabetes care regimen. What statement by the nurse is most appropriate considering common insulin therapy problems? a. Increasing your insulin levels is a good idea to lower your glucose b. How many times a week does this happen? c. This is a side effect of insulin therapy and it is called lipodystrophy d. Hyperglycemia in the AM is a problem related with insulin therapy and the doctor must assess the cause of hyperglycemia before making changes to your insulin therapy.

D. Hyperglycemia in the AM is a problem related with insulin therapy and the doctor must assess the cause of hyperglycemia before making changes to your insulin therapy. Hyperglycemia in the morning may be due to the Somogyi effect or the Dawn phenomenon. Careful assessment is required to document the Somogyi effect or dawn phenomenon because the treatment for each differs. The treatment for Somogyi effect is using less insulin in the evening. The treatment for dawn phenomenon is to increase the amount of insulin or adjust the administration time.

A 42-year-old female has recently been diagnosed with acromegaly. The patient may experience all of the following EXCEPT: a. Visual changes b. Tongue enlargement c. Crippling joint pain d. Longer bones in arms and legs

D. Longer bones in arms and legs The problem develops after epiphyseal closure, the bones of the arms and legs do NOT grow longer.

A patient is admitted to the hospital with a ventricular septal rupture. He is experiencing tachycardia, tachypnea, crackles, and a narrowing pulse pressure. The patient is suspected to be in cardiogenic shock. The nurse would question which medication ordered? a. Nitroglycerin b. Dobutamine c. Furosemide d. Norepinephrine e. Dopamine

D. Norepinephrine The nurse would question the order for norepinephrine because it is a sympathomimetic drug that increases SVR. An increase in SVR increases the workload of the heart and can cause further myocardial damage to a patient in cardiogenic shock.

During an hourly neurological assessment of a patient suffering from neurogenic shock, which of the following signs or symptoms would the nurse find most concerning? a. Delirium b. Restlessness c. Decreased responsiveness to stimuli d. Pupils nonreactive and dilated

D. Pupils nonreactive and dilated Pupils nonreactive and dilated indicate progression into the final stage of shock. The refractory stage of neurogenic shock is characterized by unresponsiveness, loss of reflexes, and nonreactive and dilated pupils.

Which manifestation would be MOST concerning to the nurse in a patient with a suspected diagnosis of acute post streptococcal glomerulonephritis? a. Edema b. Proteinuria c. Hematuria d. Sore throat e. Hypertension

D. Sore throat Sore throat and skin lesions are the main concern when determining the diagnosis. Early diagnosis and treatment with antibiotics of sore throat and skin lesions with positive streptococci is key in preventing acute post streptococcal glomerulonephritis.

The insulin regiment that most closely mimics endogenous insulin production is the basal-bolus regimen. Which of the following types of insulin is used as a basal insulin to maintain blood glucose levels in between meals and overnight? a. Lispro (Humalog) b. Aspart (Novolog) c. Regular (Humulin R) d. Glulisine (Apidra) e. Glargine (Lantus)

E. Glargine (Lantus) Glargine is a long-acting (basal) background insulin that releases steadily and continuously and does not have a peak of action which reduces the risk of hypoglycemia episodes. This type of insulin is used to maintain blood glucose levels between meals and overnight.


Kaugnay na mga set ng pag-aaral

Histology: Chaper 6- Adipose Tissue

View Set

Solid, Liquid, Gas, Freezing, Melting, Evaporation, Boiling, Vaporization, Condensation, Sublimation

View Set

Ch. 12 Critical Appraisal of Quantitative and Qualitative Research for Nursing Practice

View Set